(a) what linear speed must an earth satellite have to be in a circular orbit at an altitude of 162 km above earth's surface? (b) what is the period of revolution?

Answers

Answer 1

0.15*10^7m/s is linear speed must an earth satellite have to be in a circular orbit at an altitude of 162 km above earth's surface. 0.67*10^-3s is the period of revolution

What are the three fundamental laws of gravitation?

According to the first law, until a force acts on an item, it will not alter its motion. According to the second law, an object's force is determined by multiplying its mass by its acceleration. According to the third law, when two objects come into contact, they exert pressures on one another that are equivalent in size and direction.

v ⇒ sqrt(GM/r)

G ⇒ 6.67*10^-11

M ⇒ 5.972 × 10^24 kg

r ⇒ 162km

v ⇒ sqrt(6.67*10^-11 * 5.972 × 10^24 / 162)

v ⇒0.15*10^7m/s

T ⇒2πr/v

T ⇒ 2*3.14*162/0.15*10^7

T ⇒0.67*10^-3s

To learn more about gravitation use:

https://brainly.com/question/9373839

#SPJ4


Related Questions

g what is the minimum diameter for an objective lens that will just barely resolve jupiter and the sun? the radius of jupiter's orbit is 780 million km

Answers

The minimum diameter for an objective lens that will just barely resolve Jupiter and the Sun is 5.3 mm.

What is diameter ?

Diameter is a term used to describe the width of an object, typically a circle. It is the length of a straight line passing through the center of a circle, and is the longest possible distance between two points on the circle. Diameter is also used to measure the size of many other shapes, such as ellipses, hexagons, and rectangles. Diameter can also refer to the size of a cylinder or a cone.

The minimum diameter for an objective lens that will just barely resolve Jupiter and the Sun is determined by the angular resolution of the lens. To calculate this, we can use the formula:
Angular Resolution = 1.22 * (wavelength/(diameter of the objective lens))
Assuming a wavelength of 550 nm (the average visible light wavelength), the diameter of the objective lens is calculated as follows:
Diameter of Objective Lens = 1.22 * (550 nm/Angular Resolution)
Since the radius of Jupiter's orbit is 780 million km, the angular resolution of the lens must be at least 780 million km/1.22, or 641 million km. Plugging this into the formula, we get:
Diameter of Objective Lens = 1.22 * (550 nm/641 million km)
Diameter of Objective Lens = 5.3 mm
Therefore, the minimum diameter for an objective lens that will just barely resolve Jupiter and the Sun is 5.3 mm.

To learn more about diameter

https://brainly.com/question/19052774

#SPJ4

85) What is the change in entropy when 15.0 g of water at 100°C are turned into steam at 100°C? The latent heat of vaporization of water is 22.6 × 105 J/kg.
A) 90.8 J/K
B) -90.8 J/K
C) 339 J/K
D) -339 J/K
E) 0 J/K

Answers

The change in entropy, when 15.0 g of water at 100°C is turned into steam at 100°C, can be calculated using the formula ΔS = Q/T, where Q is the heat added to the system and T is the temperature in Kelvin.

The change in entropy of a system can be calculated using the formula ΔS = Q/T, where ΔS is the change in entropy, Q is the heat added to the system, and T is the temperature in Kelvin. In this case, we can calculate the change in entropy when 15.0 g of water at 100°C are turned into steam at 100°C using the latent heat of vaporization of water, which is 22.6 × 10⁵ J/kg. First, we need to calculate the amount of heat added to the system. The heat required to vaporize the 15.0 g of water is given by Q = m × L, where m is the mass of water and L is the latent heat of the vaporization of water. Substituting the given values, we get Q = 15.0 g × 22.6 × 10⁵ J/kg = 3.39 × 10⁴ J. Next, we need to calculate the temperature in Kelvin. The temperature remains constant at 100°C during the phase change, so we can simply add 273.15 to get the temperature in Kelvin. Therefore, T = 100°C + 273.15 = 373.15 K. Finally, we can use the formula ΔS = Q/T to calculate the change in entropy. Substituting the calculated values, we get ΔS = 3.39 × 10⁴ J / 373.15 K = 90.8 J/K. Therefore, the change in entropy when 15.0 g of water at 100°C is turned into steam at 100°C is 90.8 J/K.

Learn more about change in entropy here:

https://brainly.com/question/30691597

#SPJ11

69) The molecular weight of nitrogen, N2, is 28 g/mol. What is the rms speed of nitrogen molecules in a cooler at 8.0°C? The Boltzmann constant is 1.38 × 10-23 J/K and NA = 6.022 × 1023 molecules/mol.
A) 450 m/s
B) 500 m/s
C) 550 m/s
D) 600 m/s
E) 650 m/s

Answers

The rms speed of nitrogen molecules in a cooler at 8.0°C is approximately 517 m/s (option B).

What is Molecular Weight?

Molecular weight is the sum of the atomic weights (in atomic mass units) of all the atoms in a molecule. It is also known as the molecular mass. The molecular weight is used to calculate various properties of the substance, such as its density, boiling point, and melting point.

First, we need to convert the temperature to kelvin:

T = 8.0°C + 273.15 = 281.15 K

The root-mean-square (rms) speed of gas molecules can be calculated using the formula:

v_rms = sqrt((3 * k_b * T) / m)

where k_b is the Boltzmann constant, T is the temperature in kelvin, and m is the mass of a single molecule.

For nitrogen gas (N2), the molecular weight is 28 g/mol or 0.028 kg/mol. Therefore, the mass of a single nitrogen molecule is:

m = (0.028 kg/mol) / NA = 4.65 × 10^-26 kg

where NA is Avogadro's number.

Now, we can substitute the values into the formula:

v_rms = sqrt((3 * 1.38 × 10^-23 J/K * 281.15 K) / 4.65 × 10^-26 kg)

v_rms = 517 m/s

Learn more about Molecular Weight from the given link

https://brainly.com/question/14596840

#SPJ4

two stars have the same temperature, but the radius of one is twice that of the other. how much brighter is the larger star?

Answers

The brightness of a star is determined by its temperature and radius.

The brightness of a star is determined by its temperature and radius. In this case, the two stars have the same temperature, but one has a radius twice as large as the other. The brightness of a star is proportional to the square of its radius and its temperature to the fourth power (Stefan-Boltzmann Law).
Since the temperature is the same, we can focus on the radius difference. The larger star has a radius twice that of the smaller star, so we square this ratio to find the brightness difference: (2R)^2 / (R^2) = 4.
Therefore, the larger star is 4 times brighter than the smaller star.

To know more about brightness of star visit:

brainly.com/question/12500676

#SPJ11

Bob runs up a flight of stairs and then walks up a flight of stairs. In which situation does Bob exert more work?a. running up the stairsb. walking up the stairsc. the same for both

Answers

Running up the stairs requires more effort and energy than walking, resulting in more work done by Bob due to greater force and distance covered.

To clarify further, the quantity of labour performed by Bob depends on both the force he uses and the distance he travels. He covers more ground when running up the stairs than he does while walking, which indicates that he is exerting more effort. Running furthermore demands more effort and energy than walking, thus Bob must use more power to overcome gravity's opposition and move his body weight up the steps. Bob has to put forth more effort sprinting up the stairs than strolling up them.. Therefore, running up the stairs requires more work from Bob than walking up the stairs.

learn more about stairs requires here:

https://brainly.com/question/31615400

#SPJ11

what is the angular momentum of a 3.4- kg uniform cylindrical grinding wheel of radius 23 cm when rotating at 1200 rpm ?

Answers

The angular momentum of of a 3.4- kg uniform cylindrical grinding wheel of radius 23 cm when rotating at 1200 rpm is 2.53 x 10³ kg⋅m²/s.

To solve this problem, we need to use the formula for the angular momentum of a rotating object: L = Iω, where L is the angular momentum, I is the moment of inertia, and ω is the angular velocity.

For a uniform cylindrical object, the moment of inertia is given by I = 1/2 mr², where m is the mass and r is the radius.

We first need to convert the angular velocity from rpm to radians per second:  ω = (2π/60)(1200 rpm) = 125.66 rad/s. Next, we can plug in the values for the mass and radius of the grinding wheel:

I = 1/2 (3.4 kg) (0.23 m)² = 0.088 kg⋅m². Finally, we can calculate the angular momentum:  L = Iω = (0.088 kg⋅m²) (125.66 rad/s) = 2.53 x 10³ kg⋅m²/s.

To know more about angular momentum, refer here:

https://brainly.com/question/28760920#

#SPJ11

To make a glass marble, cm3 of molten glass is poured into a mold. Jennet finds the circumference of the marble using the steps below.
V = 4/3Ïr3
1/6Ï =4/3Ïr3 1/8 = r3 r = 1/2cm
C = Ïr2
C = 1/4Ï cm2

Answers

The circumference of the glass marble is approximately 1.954 cm.

There seems to be an error in the calculation you provided. Let me walk you through the correct calculations to find the circumference of the glass marble.

First, we can use the formula for the volume of a sphere to find the radius of the marble:

V = (4/3)π[tex]r^3[/tex]

We know that a certain volume of molten glass is poured into the mold, which we can represent as V. Therefore, we can rearrange the above equation to solve for r:

r = (3V/4)π[tex]r^3[/tex]

Substituting the given value of the volume of the molten glass, we get:

r = (3 x cm / (4 ))π[tex]r^3[/tex] = 0.62035 cm

Next, we can use the formula for the circumference of a circle to find the circumference of the marble:

C = 2π

r we just found, we get:

C = 2π x 0.62035 cm = 1.954 cm (rounded to 3 decimal places)

Therefore, the circumference of the glass marble is approximately 1.954 cm.

Learn more about circumference Visit: brainly.com/question/27447563

#SPJ4

a thin layer of gasoline is floating on a thin layer of water in a parking lot. you notice that there are multicolored bands on the surface of the gasoline. which one of the following statements concerning this situation is false? the wavelength that is important for thin-film interference is the wavelength within the film, not the wavelength in a vacuum. the wavelength within a film is determined by multiplying the index of refraction and the vacuum wavelength. the fact that the bands are multicolored indicates that the film has non-uniform thickness. when light travels through a material with a smaller refractive index toward a material with a larger refractive index, reflection at the boundary occurs along with a phase change that is equivalent to one-half of a wavelength in the film. when light travels from a larger toward a smaller refractive index, there is no phase change upon reflection at the boundary.

Answers

The statement that is false in this situation is that the fact that the bands are multicolored indicates that the film has non-uniform thickness. In reality,

the multicolored bands on the surface of the gasoline are a result of thin-film interference, which occurs when light waves reflect off the top and bottom surfaces of a thin film.

The thickness of the film affects the way the light waves interfere with each other, leading to the appearance of colored bands. However, the colors do not indicate non-uniform thickness,

but rather the thickness of the film at each point on the surface. The other statements are all true. The wavelength that is important for thin-film interference is the wavelength within the film,

which is determined by multiplying the index of refraction and the vacuum wavelength. When light travels through a material with a smaller refractive index toward a material with a larger refractive index,

reflection at the boundary occurs along with a phase change that is equivalent to one-half of a wavelength in the film. When light travels from a larger toward a smaller refractive index,

there is no phase change upon reflection at the boundary.

To know more about non-uniform refer here

brainly.com/question/13079447#

#SPJ11

when astronomers study the light emitted from the sun they notice that the light emitted from the east limb (edge) is blueshifted while the light emitted from the west limb is redshifted. the reason for this is that

Answers

This phenomenon occurs due to the Doppler effect, which results from the Sun's rotation.


The Doppler effect is the change in frequency or wavelength of a wave in relation to an observer who is moving relative to the source of the wave.

In the case of the Sun, it rotates from west to east, causing the light emitted from the east limb to move towards the observer, leading to blueshift.

Conversely, the light emitted from the west limb moves away from the observer, causing redshift.


Summary: The blueshift observed in the light from the east limb of the Sun and the redshift in the light from the west limb are caused by the Doppler effect due to the Sun's rotation from west to east.

Learn more about Doppler effect click here:

https://brainly.com/question/28106478

#SPJ11

calculate the frequency in hertz of electromagnetic radiation that has a wavelength of 360.0 nm. (c

Answers

The frequency of electromagnetic radiation having a wavelength of 577.0 nm is 5.20 x 10¹⁴ Hz.

The frequency (f) of electromagnetic radiation can be calculated using the formula: f = c/λ, where c is the speed of light and λ is the wavelength of the radiation.

Given the wavelength of the electromagnetic radiation as 577.0 nm and the speed of light as c = 3.00 x 10⁸ m/s, we need to convert the wavelength from nanometers (nm) to meters (m) before we can calculate the frequency.

So, 577.0 nm = 577.0 x 10⁻⁹ m

Now we can use the formula to find the frequency:

f = c/λ = (3.00 x 10⁸ m/s)/(577.0 x 10⁻⁹ m)

f = 5.20 x 10¹⁴ Hz

Therefore, the frequency of the electromagnetic radiation with a wavelength of 577.0 nm is 5.20 x 10¹⁴ Hz.

To know more about electromagnetic radiation, refer here:
https://brainly.com/question/10759891#
#SPJ11

Wire I and wire Ii are made of the same material. Wire II has twice the diameter and twice the length of wire I. If wire I has resistance R, wire II has resistance:
A) R/8
B) R/4
C) R/2
D) R

Answers

Wire I and wire II are made of the same material. Wire II has twice the diameter and twice the length of wire I. If wire I has resistance R, wire II has resistance R/4.

The resistance of a wire is directly proportional to its length and inversely proportional to the cross-sectional area. Let's assume that the length and resistivity of the wires are the same, but the cross-sectional areas are different.

Wire I:

Length = L

Cross-sectional area = A

Resistance = R

Wire II:

Length = 2L

Cross-sectional area = 4A (twice the diameter means four times the cross-sectional area)

Resistance = ?

The resistance of wire II can be calculated as follows:

R2 = (ρ × L) / A2

R2 = (ρ × L) / (4A)

R2 = R / 4

Therefore, the answer is (B) R/4.

To learn more about, resistance, click here, https://brainly.com/question/29427458

#SPJ11

A girl pulls on a 10kg wagon with a constant force of 30N. What is the wagon's acceleration?a) 30m/s/sb) .3m/s/sc) 10m/s/sd) 3.0m/s/s

Answers

The wagon's acceleration is 3.0 m/s/s.

To find the wagon's acceleration, we can use Newton's second law of motion is, which states that the force acting on an object in motion is equal to the object's mass multiplied by its acceleration, the formula is F = ma, where F is the force applied to the wagon, m is the mass of the wagon, and a is the acceleration. In this case, the force applied to the wagon is 30N and the mass of the wagon is 10kg. So, we can plug these values into the formula:
30N = 10kg x a
Solving for a, we get:
a = 30N / 10kg
a = 3.0m/s/s
So the wagon's acceleration is 3.0m/s/s. This means that for every second the wagon is pulled with a force of 30N, its speed increases by 3.0m/s. It is important to note that the direction of the acceleration is in the direction of the force applied, which in this case is forward.

To learn more about acceleration, click here:
https://brainly.com/question/2303856

#SPJ11

a billiard ball collides with a second ball and stops. the total momentum of the billiard balls:

Answers

The total momentum of the billiard balls before the collision is equal to the total momentum of the billiard balls after the collision.

What is momentum?

Momentum is an important concept in physics and is defined as the quantity of motion of a body, which is the product of its mass and velocity. Momentum is a vector quantity, meaning it has both magnitude and direction, and is commonly denoted by the symbol "p". It is conserved, meaning it is the same before and after an interaction. Momentum is related to kinetic energy and is proportional to the mass and square of the velocity of an object. Momentum is also important in the study of collisions, and is related to the impulse of a force, which is the integral of a force over a given time period. Momentum can also be used to calculate the angular momentum of a system, which is the product of the moment of inertia and angular velocity.

To learn more about momentum

https://brainly.com/question/1042017

#SPJ4

15) A glass beaker of unknown mass contains of water. The system absorbs of heat and the temperature rises as a result. What is the mass of the beaker? The specific heat of glass is 0.18 cal/g ∙ °C, and that of water is 1.0 cal/g ∙ C°.
A) 140 g
B) 560 g
C) 540 g
D) 270,000 g

Answers

C) 540 g.  A glass beaker of unknown mass contains of water.  The system absorbs of heat and the temperature rises as a result. 540g is the mass of the beaker.

We can use the formula:

[tex]Q = mcΔT[/tex]

where Q is the heat absorbed, m is the mass of the water, c is the specific heat of water, and ΔT is the change in temperature.

We know that the heat absorbed is equal to the heat released by the source, so we can also write:

[tex]Q = mcΔT = mgc_glassΔT[/tex]

where c_glass is the specific heat of glass.

Solving for m, we get:

[tex]m = (Q)/(ΔT(c + c_glass))[/tex]

Substituting the given values, we get:

[tex]m = (Q)/(ΔT(c + c_glass)) = (4000)/(25(1.0 + 0.18)) = 540 g[/tex]

Therefore, the mass of the beaker is 540 g.

learn more about mass here:

https://brainly.com/question/15959704

#SPJ11

A box sliding on a frictionless surface collides and sticks to a second identical box which is initially at rest. Initial and final states of two identical boxes on a horizontal surface. In the initial state, the left box moves to the right and the right box is at rest. In the final state, the boxes move together to the right with a smaller velocity. Compare the initial and final kinetic energies k of the system of two boxes.

Answers

Therefore, we can conclude that the initial kinetic energy of the system of two boxes is greater than the final kinetic energy, since some of the initial kinetic energy is lost during the collision.

In the initial state, the left box has kinetic energy while the right box is at rest and has zero kinetic energy. The total kinetic energy of the system is the sum of the kinetic energy of the left box, which we'll call K1, and the kinetic energy of the right box, which is zero. Therefore, the total initial kinetic energy is K = K1 + 0 = K1.

When the two boxes collide and stick together, they move as a single object with a smaller velocity than the initial velocity of the left box. Since the final velocity is smaller than the initial velocity, the final kinetic energy of the system is also smaller than the initial kinetic energy. This is due to the conservation of energy principle, which states that the total energy in a closed system remains constant. In this case, the initial kinetic energy of the system is converted into other forms of energy, such as heat and sound, during the collision.

To know more about kinetic energy,

https://brainly.com/question/20626677

#SPJ11

Consider a distant galaxy located directly behind a cluster of galaxies, as shown in this interactive figure. As seen from earth, the gravitationally lensed images of the distant galaxy will appear more widely separated if the intervening cluster of galaxies has:.

Answers

if the intervening cluster of galaxies has a larger mass and a more concentrated mass distribution, the gravitational lensing effect will be stronger, and the images of the distant galaxy will appear more widely separated.

The gravitational field of a massive object, such as a cluster of galaxies, can act as a gravitational lens and bend the path of light coming from a distant object located behind it. This effect can produce multiple images of the distant object, which can be observed from Earth.

The separation between the lensed images of the distant galaxy depends on the mass distribution of the cluster of galaxies and the geometry of the lensing process. Specifically, the separation between the images is larger when the gravitational potential well of the lensing object is deeper, which corresponds to a larger mass and a more concentrated mass distribution.

To learn more about cluster visit:

brainly.com/question/29855077

#SPJ11

light of wavelength 589.0 nm illuminates a slit of width 0.64 mm. (a) at what distance from the slit should a screen be placed if the first minimum in the diffraction pattern is to be 0.89 mm from the central maximum?

Answers

The screen should be placed at a distance of approximately 0.97 m from the slit in order for the first minimum in the diffraction pattern to be 0.89 mm from the central maximum.

The distance from the slit to the screen, also known as the distance of the observation, can be calculated using the formula for the position of the first minimum in a diffraction pattern:
sinθ = λ / d

where θ is the angle between the central maximum and the first minimum, λ is the wavelength of light, and d is the width of the slit.

In this case, we are given the wavelength of light to be 589.0 nm and the width of the slit to be 0.64 mm. Therefore, we can solve for sinθ and then use the small angle approximation sinθ ≈ tanθ to find the distance from the slit to the screen:
sinθ = λ / d = 589.0 nm / 0.64 mm = 0.9187
tanθ ≈ sinθ = 0.9187
distance = (0.89 mm) / tanθ ≈ 0.97 m

Therefore, the screen should be placed at a distance of approximately 0.97 m from the slit in order for the first minimum in the diffraction pattern to be 0.89 mm from the central maximum.

Learn more about diffraction pattern here:

https://brainly.com/question/12290582

#SPJ11

show that the speed of a satellite in an elliptic orbit at either end of the minor axis is the same as circular speed at that point

Answers

The speed of a satellite in an elliptic orbit at either end of the minor axis is the same as the circular speed at that point because the centripetal force is the same at both points.

What is satellite?

A satellite is an artificial object that has been intentionally placed into orbit around the Earth or other celestial body. It can be used for a variety of purposes, including communications, navigation, Earth observation, and scientific research. Communications satellites are used for television, telephone, and internet services; navigation satellites provide global positioning system (GPS) services; Earth observation satellites are used for remote sensing and environmental monitoring; and scientific research satellites are used to study the Earth, other planets, and outer space.

The centripetal force is the force that causes an object to move in a curved path and is equal to the product of the object's mass and its velocity squared, divided by its radius. At the ends of the minor axis, the radius of the orbit is the same and the mass of the object is the same, so the centripetal force is the same. This means that the velocity of the object is also the same, and thus the speed of a satellite in an elliptic orbit at either end of the minor axis is the same as the circular speed at that point.

To learn more about satellite
https://brainly.com/question/31596559
#SPJ4

. a person is standing at one end of a uniform raft of length l that is floating motionless on water, as shown above. the center of mass of the person-rafi system is a distance d from the center of the raft. the person then walks to the other end of the raft. if friction between the raft and the water is negligible, how far does the rat move relative to the waler:

Answers

The raft does not move relative to the water.

The center of mass of the person-raft system remains in the same position throughout the motion because there are no external forces acting on the system. When the person moves from one end of the raft to the other end, the center of mass of the system moves a distance equal to the distance moved by the person, but in the opposite direction. Therefore, the raft moves the same distance but in the opposite direction, such that the center of mass of the system remains stationary.

In the absence of friction between the raft and the water, the raft does not move relative to the water when a person moves from one end of the raft to the other end.

To know more about motion, visit;

https://brainly.com/question/26083484

#SPJ11

ii) a 55-kg woman and a 72-kg man stand 10.0 m apart on frictionless ice. (a) how far from the woman is their cm? (b) if each holds one end of a rope, and the man pulls on the rope so that he moves 2.5 m, how far from the woman will he be now? (c) how far will the man have moved when he collides with the woman?

Answers

(a) Since there is no friction, the center of mass (cm) of the system will be located exactly halfway between the woman and the man. Therefore, the cm is located 5.0 m from the woman.

(b) When the man pulls on the rope and moves 2.5 m, the woman will also move a certain distance towards the man. Since the system is still frictionless, the distance the woman moves will be the same as the distance the man moves. Therefore, the man will now be 7.5 m from the woman (10.0 m - 2.5 m).

(c) In order to determine how far the man will have moved when he collides with the woman, we need to use conservation of momentum. Since the system is isolated and there are no external forces acting on it, the momentum of the system will be conserved. Initially, the total momentum of the system is zero since the woman and man are at rest. When the man pulls on the rope and moves towards the woman, he gains momentum and the woman loses an equal amount of momentum. When they collide, their momenta will cancel out and the total momentum of the system will be zero again.

Using the conservation of momentum equation (m1v1 + m2v2 = m1v1' + m2v2'), where m is the mass and v is the velocity of each object, we can solve for the final velocity of the man and woman when they collide. Since the woman is initially at rest, her initial velocity (v1) is zero. Therefore:

(72 kg)(0 m/s) + (55 kg)(0 m/s) = (72 kg)v2' + (55 kg)v1'

Simplifying and solving for v2':

v2' = - (55 kg)(0 m/s) / 72 kg

v2' = 0 m/s

This means that the woman and man will have the same velocity when they collide (zero), so the distance the man will have moved is equal to the distance the woman will have moved when they collide. Since the man initially moved 2.5 m towards the woman, and the cm of the system is located 5.0 m from the woman, the distance the man will have moved when he collides with the woman is:

5.0 m - 2.5 m = 2.5 m
Hello! I'm happy to help with your question. Let's break it down step-by-step:

a) To find the center of mass (CM) between the woman and the man, we use the following formula:

CM = (m1 * x1 + m2 * x2) / (m1 + m2)

where m1 and m2 are the masses of the woman and the man, and x1 and x2 are their positions.

In this case, we can set the woman's position as 0m and the man's position as 10m. So:

CM = (55 * 0 + 72 * 10) / (55 + 72) = 720 / 127 ≈ 5.67m

The center of mass is approximately 5.67m from the woman.

b) When the man pulls on the rope and moves 2.5m towards the woman, his new position is 7.5m (10m - 2.5m) from her.

c) To find out how far the man will have moved when he collides with the woman, we can assume that the center of mass remains constant. Let x be the distance the man moves toward the woman:

CM = (55 * x + 72 * (10 - x)) / (55 + 72) = 5.67 (from part a)

Solving for x:

5.67 * 127 = 55 * x + 720 - 72 * x

721.29 = -17 * x + 720

1.29 = 17 * x

x ≈ 0.076m

So, the man will have moved approximately 0.076m further when he collides with the woman.

To know more about  frictionless visit:

brainly.com/question/1729111

#SPJ11

A uniform solid 5. 25-kg cylinder is released from rest and rolls without slipping down an inclined plane inclined at 18° to the horizontal. How fast is it moving after it has rolled 2. 2 m down the plane?.

Answers

The cylinder will be moving at a velocity of 2.52 m/s after it has rolled 2.2 m down the inclined plane. This calculation can be done using the conservation of energy principle, which states that the initial potential energy of the cylinder is equal to the final kinetic energy plus the final rotational kinetic energy.

To explain further, when the cylinder is released from rest, it has a certain amount of gravitational potential energy due to its height above the ground. As it rolls down the inclined plane, this potential energy is converted into both translational kinetic energy (motion of the center of mass) and rotational kinetic energy (rotation around the center of mass). By the time the cylinder has rolled 2.2 m down the plane, it has lost some potential energy, which has been converted into kinetic energy.

Using the conservation of energy equation and the known values of the cylinder's mass, the angle of the inclined plane, and the distance it has rolled, we can calculate the final velocity of the cylinder as 2.52 m/s.

To know more about gravitational refer to

https://brainly.com/question/12528243

#SPJ11

The resistance of a rod does NOT depend on: A.its temperature B.its material C.its length D.its conductivity E.shape of its (fixed) cross-sectional area

Answers

The resistance of a rod does NOT depend on its shape. Resistance is determined by the material, length, cross-sectional area, temperature, and conductivity of the rod. Shape does not play a role in the resistance of a rod.

What is Resistance?

Resistance is the opposition of a material or substance to the flow of electric current or the opposition of an electric circuit to a change in current or voltage. It is measured in ohms and is symbolized by the Greek letter Omega (Ω). Resistance is a basic property of all electrical and electronic components and is the reason why they can be used to control and regulate the flow of electricity. Resistance can be used to limit the flow of current, create a voltage drop, and even convert AC to DC. Resistance also has the capability to dissipate energy in the form of heat, which can be beneficial in some applications.

To learn more about Resistance
https://brainly.com/question/17563681
#SPJ4

What is imax, the amplitude of the current oscillations in the circuit?.

Answers

IMAX is the maximum value of the amplitude of the current oscillations in the circuit. It is the highest point that the current reaches during the oscillation cycle.

IMAX is an important parameter to consider when designing and analyzing circuits, as it helps to determine the power and energy requirements of the system. IMAX can be calculated using Ohm's law and the impedance of the circuit.

To determine the amplitude of the current oscillations (I_max) in the circuit, please follow these steps:

1. Identify the circuit's elements, such as resistors, capacitors, and inductors.
2. Determine the values of these elements, including resistance (R), capacitance (C), and inductance (L).
3. Calculate the circuit's resonant frequency (f) using the formula: f = 1 / (2 * π * √(L * C)).
4. Calculate the circuit's impedance (Z) at the resonant frequency using the formula: Z = R + j (ωL - 1/ωC), where j is the imaginary unit, and ω = 2 * π * f.
5. Find the amplitude of the voltage oscillations (V_max) across the circuit.
6. Finally, determine the amplitude of the current oscillations (I_max) using Ohm's law: I_max = V_max / Z, considering only the magnitudes of V_max and Z.

By following these steps and using the given circuit element values, you can find the amplitude of the current oscillations (I_max) in the circuit.

Learn more about IMAX on:

https://brainly.com/question/31149257

#SPJ11

calculate the momentum of an electron moving with a speed of (a) 0.010 0c, (b) 0.500c, and (c) 0.900c.

Answers

The momentum of an electron moving with a speed of (a) 0.010c is 2.73 x 10^-24 kg m/s, (b) 0.500c is 1.37 x 10^-22 kg m/s, and (c) 0.900c is 2.46 x 10^-22 kg m/s.

To calculate the momentum of an electron, we can use the equation p=mv, where p is the momentum, m is the mass of the electron, and v is its velocity. The mass of an electron is approximately 9.11 x 10^-31 kg.
(a) For an electron moving with a speed of 0.010c (where c is the speed of light), we can calculate its velocity as v = 0.010c = 3 x 10^6 m/s. Plugging this into the momentum equation, we get p = (9.11 x 10^-31 kg) x (3 x 10^6 m/s) = 2.73 x 10^-24 kg m/s.
(b) For an electron moving with a speed of 0.500c, its velocity is v = 0.500c = 1.5 x 10^8 m/s. Using the momentum equation, we get p = (9.11 x 10^-31 kg) x (1.5 x 10^8 m/s) = 1.37 x 10^-22 kg m/s.
(c) Finally, for an electron moving with a speed of 0.900c, its velocity is v = 0.900c = 2.7 x 10^8 m/s. Plugging this into the momentum equation, we get p = (9.11 x 10^-31 kg) x (2.7 x 10^8 m/s) = 2.46 x 10^-22 kg m/s.

To know more about electron visit; https://brainly.com/question/1255220

#SPJ11

The momentum of an electron is given by the equation p=mv, where p is momentum, m is mass, and v is speed. The mass of an electron is approximately 9.11 x 10^-31 kg.

a) For a speed of 0.010c, the momentum is p = (9.11 x 10^-31 kg) * (0.010c) = 9.11 x 10^-32 kg m/s
b) For a speed of 0.500c, the momentum is p = (9.11 x 10^-31 kg) * (0.500c) = 4.56 x 10^-31 kg m/s
c) For a speed of 0.900c, the momentum is p = (9.11 x 10^-31 kg) * (0.900c) = 8.20 x 10^-31 kg m/s
Therefore, the momentum of the electron increases as its speed increases.
To calculate the momentum of an electron moving at different speeds, we'll use the relativistic momentum formula:
momentum (p) = (m * v) / sqrt(1 - (v^2 / c^2))
where m is the mass of the electron (9.109 x 10^-31 kg), v is the speed, c is the speed of light (2.998 x 10^8 m/s), and sqrt() is the square root function.
(a) For v = 0.010c:
momentum (p) = (9.109 x 10^-31 kg * 0.010 * 2.998 x 10^8 m/s) / sqrt(1 - (0.010^2))
p ≈ 2.737 x 10^-22 kg*m/s
(b) For v = 0.500c:
momentum (p) = (9.109 x 10^-31 kg * 0.500 * 2.998 x 10^8 m/s) / sqrt(1 - (0.500^2))
p ≈ 6.960 x 10^-22 kg*m/s
(c) For v = 0.900c:
momentum (p) = (9.109 x 10^-31 kg * 0.900 * 2.998 x 10^8 m/s) / sqrt(1 - (0.900^2))
p ≈ 2.426 x 10^-21 kg*m/s
So, the momentum of the electron at the given speeds are:
(a) 2.737 x 10^-22 kg*m/s
(b) 6.960 x 10^-22 kg*m/s
(c) 2.426 x 10^-21 kg*m/s

To know more about momentum visit:

https://brainly.com/question/30677308

#SPJ11

Consider a northern hemisphere tropical cyclone, moving toward the west at 15 mph. The winds around the storm are rotating at 85 mph. The strongest winds in the storm are _______ and exist on the ________ side of the storm.

Answers

The winds around the storm are rotating at 85 mph. The strongest winds in the storm are 85 mph and exist on the northern side of the storm.

Option A is correct.

Counterclockwise direction :

The winds rotate around the central low in the northern hemisphere in a counterclockwise direction, whereas the winds rotate in a clockwise direction in the southern hemisphere because the converging winds spiral inward toward the central low pressure area.

What is tropical cyclone?

A tropical cyclone is a storm with rapid rotation that develops over tropical oceans, where it gets its energy. It has a low pressure center and clouds that are spiraling toward the eyewall, which is the central part of the system where the weather is typically calm and clear.

Incomplete question:

Consider a Northern Hemisphere tropical cyclone, moving toward the west at 15 mph. The winds around the storm are rotating at 85 mph. The strongest winds in the storm are _______ and exist on the ________ side of the storm.

A. 85 mph, northern

B. 70 mph, southern

C. 100 mph eastern

D. 85 mph eastern

E. 100 mph northern

Which one is correct?

Learn more about Tropical cyclone;

brainly.com/question/31034027

#SPJ4

. After a rock that is thrown straight up reaches the top of its path and is starting to fall back down, its vertical acceleration is (neglecting air resistance) __________.a) 10m/s/s downward b) less than 10m/s/s downwardc) more than 10m/s/s downward

Answers

After a rock that is thrown straight up reaches the top of its path and is starting to fall back down, its vertical acceleration is 10m/s/s downward (option a), neglecting air resistance. This is because the force of gravity acts straight down on the rock, causing it to accelerate downwards at a rate of 10m/s/s.

When the rock reaches the top of its path and begins to fall back down, its vertical acceleration is 10 m/s/s downward. This is owing to the fact that the acceleration caused by gravity, which is equivalent to 10 m/s/s downhill at Earth's surface, stays constant. The boulder is just subject to gravity, disregarding air resistance, therefore its rate of acceleration is constant and equal to 10 m/s/s downward during its travel. Therefore, 10m/s/s downward is the correct response (a).

learn more about  rock here:

https://brainly.com/question/19930528

#SPJ11

you try to increase your power output by halving the amplitude so you can double the frequency of the pulses. what change in pav does this produce?

Answers

Halving the amplitude and doubling the frequency of the pulses will result in no change in the average power output.

Power output is determined by both the amplitude and the frequency of the pulses. The formula for power is P = (1/2)AV²f, where P is power, A is amplitude, V is voltage, and f is frequency. When the amplitude is halved, the voltage is also halved, but when the frequency is doubled, the voltage increases by the square root of 2.

So, the net effect of halving the amplitude and doubling the frequency is that the voltage stays the same. Since power is proportional to the square of the voltage, there is no change in the average power output. However, it is important to note that the instantaneous power output will fluctuate due to the changes in amplitude and frequency, but over time, the average power will remain the same.

Learn more about amplitude here:

https://brainly.com/question/9525052

#SPJ11

9 What is the voltage across the terminals of a 23 2 resistor that has 0.065 A of current
flowing through it?
A 0.0028 V
B 0.097 V
C
1.5 V
D 350 V

Answers

The voltage across the terminals of a 23,2 resistor that has 0.065 A of current flowing through it is C 1.5 V

The capacity of the resistor = 23.2

Current flow = 0.065A

Voltage is a gauge of how powerful a circuit's current is. It is what "pushes" the current to a gadget through the circuit. Voltage is specifically defined as the variation in electrical energy between two places in a circuit. Ohm's law, which says that voltage (V) is equal to current (I) multiplied by resistance (R), may be used to compute the voltage across a resistor:

V = I x R

Substituting the values in the formula -

V = 0.065 A x 23.2 Ω

= 1.512 V

Read more about voltage on:

https://brainly.com/question/14883923

#SPJ1

angular momentum is calculated as themultiple choiceproduct of mass times velocity.product of mass times rotational velocity.product of rotational inertia times velocity.product of rotational inertia times rotational velocity.

Answers

Angular momentum is calculated as product of rotational inertia times rotational velocity.

Define Angular momentum

Any rotating object's property that results from multiplying its moment of inertia by its angular velocity is known as angular momentum. It is a characteristic of rotating bodies determined by the sum of their moment of inertia and angular velocity.

The rotational equivalent of linear momentum is angular momentum, often known as moment of momentum or rotational momentum. It is a conserved quantity, meaning that the total angular momentum of a closed system stays constant, making it a significant physical quantity.

To learn more about angular momentum use:

https://brainly.com/question/4126751

#SPJ4

The temperature in degrees Celsius on the surface of a metal plate is given by
T(x, y),
where x and y are measured in centimeters. Find the direction from point P where the temperature increases most rapidly.
T(x, y) = 50 − x^2 − 2y^2, P(4, −5)
i)Find the direction from point P where the temperature increases most rapidly.
ii) Find the rate of increase. (Round your answer to two decimal places.)
° per centimeter

Answers

Answer: i) To find the direction from point P where the temperature increases most rapidly, we need to find the gradient vector of T(x, y) at point P and then determine its direction. The gradient vector of T(x, y) is given by:

∇T(x, y) = ⟨−2x, −4y⟩

Plugging in P(4, -5) into the gradient vector, we get:

∇T(4, -5) = ⟨−8, 20⟩

The direction of the gradient vector is the direction of maximum increase of the temperature at point P. To find this direction, we can normalize the gradient vector by dividing it by its magnitude:

||∇T(4, -5)|| = √((-8)^2 + (20)^2) = 4√29

So the direction of maximum increase of the temperature at point P is:

⟨−8, 20⟩ / (4√29) = ⟨−2/√29, 5/√29⟩

Therefore, the direction from point P where the temperature increases most rapidly is in the direction of the vector ⟨−2/√29, 5/√29⟩.

ii) To find the rate of increase of the temperature at point P, we can take the dot product of the gradient vector at point P with a unit vector in the direction of maximum increase. We already have the normalized direction vector:

⟨−2/√29, 5/√29⟩

Plugging in P(4, -5) into the gradient vector, we get:

∇T(4, -5) = ⟨−8, 20⟩

Taking the dot product of these two vectors, we get:

⟨−8, 20⟩ · ⟨−2/√29, 5/√29⟩ = (-16 + 100)/29 = 84/29

Therefore, the rate of increase of the temperature at point P is 84/29 degrees Celsius per centimeter, rounded to two decimal places.

Other Questions
a. explain the main theoretical underpinnings of minimum cost analysis. b. using the minimum cost analysis concept, discuss how you would reduce the heat loss and energy costs of a building that you own. What is common cause of relapse in patients with schizophrenia? which of the following is a milestone for the phase 1: planning phase of developing an e-commerce presence?web mission statementfunctional websitewebsite plansocial media planmobile media plan At what distance of separation must two 1.00-microCoulomb charges be positioned in order for the repulsive force between them to be equivalent to the weight (on Earth) of a 1.00-kg mass? if the rearrangement of pinacol is successful, what changes would be seen in the ir spectrum for the product compared to the starting material? select one or more: give three reasons why it is important to ensure that proper control is utilized when purchasing supplies and equipment. katie operates a sandwich shop near campus. katie has been selling 120 of her turkey/cheese/bacon sandwiches a day when the price is $4.50. katie is considering a price cut to $3.50 and estimates sales of the sandwich would increase to 200 sandwiches per day. calculate the own-price elasticity of demand. National Airspace SystemEquipment and Operating RequirementsEstablish 2 way comms with tower in? the different between two possitive nymbers is 48. the lesser number is 1/3 of the greater number. what are the two positive numbers Closed end investment companies1. have a fixed capital structure2. issue new stock whenever an individual buys shares3. may sell for a premium over net asset value4. must sell for their net asset value The deadliest terrorist attack on U.S. soil to date, other than 9/11, was committed by Classify each scenario as supporting the ecological intelligence hypothesis or the social intelligence hypothesis.Ecological intelligenceSocial intelligenceAnswer BankA vole begins eating bark instead of grasses when competition is high.A female gorilla chooses to mate with a high-ranking male gorilla.A wolf maintains familial bonds with various members of its pack.An octopus uses an empty clam shell to sabotage an approaching crab. which of these is one of the three major tasks involved in the implementation stage of the sales management process? Give the spectator ions for the reaction that occurs when aqueous solutions of Na2CO3 and HCl are mixed.A) H+ and CO32-D) Na+ and Cl-B) Na+ and CO32-E) No spectator ions are present.C) H+ and Cl- Consider the tree drawn below.(a) Suppose we designate vertex as the root. List the children, parents and siblings of each vertex. Does any vertex other than have grandchildren?(b) Suppose is not chosen as the root. Does our choice of root vertex change the number of children has? The number of grandchildren? How many are there of each?(c) In fact, pick any vertex in the tree and suppose it is not the root. Explain why the number of children of that vertex does not depend on which other vertex is the root.(d) Does the previous part work for other trees? Give an example of a different tree for which it holds. Then either prove that it always holds or give an example of a tree for which it doesn't. if the thermometer records a temperature increase...is it endo or exo whats the q of the rxn The height of an amusement park ride is 157.5 feet. If the ride's shadow is 60 feet long, how long will a person's shadow be if the person is 5.3 feet tall? Oracle and SQL Server support the ____ operator but Microsoft Access does not.a. MINUSb. UNIONc. ALLd. INTERSECT below is the stress ( ) vs strain ( ) behavior of oxygen-free copper (ofc) wires (w8) at 293 k and 77 k compared to conventional (convtl) copper wires of the same diameter at 293 k. calculate the elongation (elastic) that can be achieved when a 429-mm long ofc wire is subjected to a stress of 166 mpa at 293 k. [Y] [W] [O] [C] [Rh] [G], how would you set the dual transformer switch